Pagina 1 di 1

Problema su cinematica (Fisica 1)

MessaggioInviato: 17/03/2019, 16:30
da Nik__
Il problema è il seguente:Si considerino due punti A e B alle due estremità di un fiume largo d in modo che il segmento AB sia perpendicolare alle sponde del fiume.L'acqua si muove con velocità di modulo pari a |v|=3.0m/s e parallela alle sponde.Un uomo nuota con velocità,rispetto all'acqua,di modulo pari a quello della velocità della corrente. Egli parte dal punto B e nuota in modo che la sua velocità punti sempre verso A.Rispetto ad un sistema di riferimento solidale con il fiume determinare: a) il modulo della velocità del nuotatore quando si trova nel puntp B;b) l'espressione delle componenti della velocità in funzione di v e tetha in un sistema di coorinate polari con l'asse polare che passa per A e B, con il polo in B.c)La traettoria del nuotatore.(Si suggerisce di usare le coordinate polari e le componenti della velocità ottenute nel punto b.Puo essere utile il seguente integrale$ int_()^()costheta d theta=log(|1/costheta+tantheta|)+C $).

Io ho pensato che se mi metto nel sistema di riferimento dell'acqua vedo l'uomo che nuota di moto circolare uniforme e percorre un quarto di circonferenza. Però sviluppando questo ragionamento mi rendo conto che non può essere qualcuno può darmi un suggerimento su come impostare il problema?Ho provato a cambiare tutti i sistemi di riferimento ma nulla.
Grazie in anticipo

Re: Problema su cinematica (Fisica 1)

MessaggioInviato: 18/03/2019, 08:03
da Palliit
Nik__ ha scritto:Puo essere utile il seguente integrale$ int_()^()costheta d theta=log(|1/costheta+tantheta|)+C $).

Sei sicuro/a che non sia: $int 1/cos vartheta dvartheta$ ?

( $int cos vartheta dvartheta =sin theta +C$ )

Re: Problema su cinematica (Fisica 1)

MessaggioInviato: 19/03/2019, 16:14
da Nik__
Si scusami ho sbagliato a scrivere

Re: Problema su cinematica (Fisica 1)

MessaggioInviato: 19/03/2019, 22:17
da mgrau
Nik__ ha scritto:Io ho pensato che se mi metto nel sistema di riferimento dell'acqua vedo l'uomo che nuota di moto circolare uniforme e percorre un quarto di circonferenza. Però sviluppando questo ragionamento mi rendo conto che non può essere

Perchè non può essere? Magari cambiando qualcosa... Non ho fatto i conti, ma, a occhio, e mettendosi nel riferimento della riva, il movimento inizia con una direzione 45° a valle, poi la direzione del nuotatore punta sempre più verso monte, con che la velocità di discesa a valle diminuisce, e diminuisce anche la velocità di avvicinamento all'altra riva. Se il nuotatore riesce a raggiungere l'altra riva, in quel momento la sua velocità è nulla: lui nuota a monte e la corrente lo porta a valle. Quindi direi che il contatto con l'altra riva avviene perpendicolarmente alla riva, così potrebbe essere un ottavo di circonferenza.
Si tratterebbe di trovare se, 1) è davvero un arco di circonferenza e 2) se davvero ci arriva

Re: Problema su cinematica (Fisica 1)

MessaggioInviato: 19/03/2019, 23:34
da ondine
Ciao! Per prima cosa penso tu abbia sbagliato a scrivere, cioè $A$ è il punto di partenza e $B$ quello d'arrivo. Prendiamo un sistema di riferimento polare come suggerito, centrato nel punto $B$. Per prima cosa scomponiamo la velocità dell'acqua nelle componenti radiali e tangenziali:
$vecV_a=-V_asinthetahatr-V_acosthetahattheta$
sappiamo inoltre che la velocità del nuotatore rispetto all'acqua è sempre puntata verso il punto $B$ e quindi ha solo la componente radiale
$vecV_n=-V_nhatr$
Per passare dal sistema di riferimento dell'acqua a quella del fiume (che ho interpretato come quello solidale al terreno) si utilizzano le trasformazioni di Galileo, cioè a parole : la velocità del nuotatore rispetto al terreno è uguale alla somma della velocità del nuotatore rispetto all'acqua e della velocità dell'acqua rispetto al terreno.
Quindi, essnedo $vecV=V_r hatr+V_theta hattheta$ le componenti radiale $V_r$ e tangenziale $V_theta$ della velocità del nuotatore rispetto al fiume sono:
$V_r=-V_n-V_asintheta$
$V_theta=-V_acostheta$
Ma $V_r=(dr)/(dt)$ e $V_theta=r(d theta)/(dt)$
quindi
$(dr)/(dt)=-V_n-V_asintheta$
$r(d theta)/(dt)=-V_acostheta$
da queste ci si ricava che
$(dr)/(d theta)=(r(V_asintheta+V_n))/(V_ncostheta)$
Sapendo che $V_a=V_n$ il tutto si semplifica e si rimane con
$(dr)/(d theta)= rtantheta+r/costheta$
intgrando
$int 1/r dr =int tantheta d theta +int1/costheta d theta$
Risolvendo gli integrali e con la condizione iniziale che $r(0)=d$ si ottiene:
$r(theta)= d((1+sintheta)/(cos^2theta))$
Il problema di questa soluzione è che il nuotatore non raggiungerà mai il punto $B$.
Infatti se volessimo esprimere tutto in cordinate cartesiane, si avrebbe che
$y=rcostheta=d((1+sintheta))/(costheta)$ e $x=rsintheta=d(sintheta(1+sintheta))/(cos^2theta)$
Se il nuotatore raggiunge il punto B vuol dire che si trova in un punto con ordinata 0.
Ma la condizione $y->0$ si traduce in $sintheta->-1 => theta-> (3pi)/2$
e in questa condizione x non tende a 0 come accadrebbe se il nuotatore finisse nel punto $B$ (l'origine), ma facendo il limite si vede che $x-> -d/2$ . Quindi il nuotatore non finisce in $B$, ma vene spostato un po' dalla corrente e finisce a $d/2$ di distanza ( e con velocità nulla).

EDIT: manipolando $y(theta)$ e $x(theta)$ si trova che la traiettoria in coordinate cartesiane è descritta dall'equazione $y(x)=sqrt(2dx+d^2)$